宏觀經濟學

有人能幫我解決穩態下的人均資本問題嗎(檢查我的工作)

  • November 29, 2020

問:假設 $ C_t=(1-s)Y_t $ 在哪裡 $ s>\sigma $ 就像在基本的索洛模型中一樣。求解穩態下的人均資本。

$ Y_t=K^{\alpha}_tL^{1-\alpha}_t $

$ Y_t=C_t+I_t+G_t $

$ K_{t+1}=I_t+(1-\delta)K_t $

$ L_{t+1}=(1+n)L_t $

$ G_t=\sigma Y_t $

試圖:

$ sy=sky^{\alpha} $ : 人均儲蓄和投資

$ (n+d)k $ :保持人均資本不變所需的投資

n:人口增長率

d:折舊

$ \dot{k}= sky{\alpha}-(n+d)k=0 $

評論:我不確定這是否正確。我的問題是我什至沒有利用上面給出的關於消費的資訊。有什麼建議可以引導我走上正確的道路嗎?

儘管他們朝著正確的方向前進,但您採取的步驟中有一些錯誤。正確的步驟如下。

首先,我們可以通過將前兩個方程除以 $ L_t $ :

$$ \frac{Y_t}{L_t}=\frac{K^{\alpha}_t L^{1−\alpha}_t}{L_t} \implies y_t = k_t^{\alpha} $$

$$ \frac{Y_t}{L_t}=\frac{C_t}{L_t}+\frac{I_t}{L_t}+\frac{G_t}{L_t} = y_t = c_t + i_t + g_t $$

其中小寫字母表示人均變數(例如 $ \frac{Y_t}{L_t} = y_t $ )。使用這個和 $ L_{t+1}=(1+n)L_t $ 按人均計算的資本演變將由下式給出:

$$ (1 + n)k_{t+1} = (1 − \delta )k_t + i_t \implies k_{t+1} \approx (1 − n -\delta )k_t + i_t $$

在這裡,我們可以使用以下事實:

$$ i_t = y_t - c_t - g_t = k_t^{\alpha} - c_t - g_t $$.

因此:

$$ k_{t+1} = (1 − n -\delta )k_t + k_t^{\alpha} - c_t - g_t $$

因此

$$ k_{t+1} - k_t= -( n + \delta )k_t + k_t^{\alpha} - c_t - g_t $$

現在自從 $ c_t=(1−s)(y_t-g_t) $ , 在哪裡 ( $ -g_t $ ) 是因為消費只取決於稅後的可支配收入和 $ g_t = \sigma y_t $ 並記住 $ y_t= k^{\delta}_t $ 我們得到:

$$ k_{t+1} - k_t = -( n + \delta )k_t + k_t^{\alpha} - ( (1−s)(k_t^{\alpha}-\sigma k_t^{\alpha})- \sigma k_t^{\alpha}) \ = -( n + \delta )k_t + k_t^{\alpha} - ( k_t^{\alpha} - s (1-\sigma) k_t^{\alpha} ) \ = s (1-\sigma) k_t^{\alpha} -( n + \delta )k_t $$

最後一個方程實際上只是這個模型版本 $ sy-(n+\delta)k_t $ . 現在終於除以 $ k_t $ 為了達到增長率,我們得到:

$$ \frac{k_{t+1} - k_t }{k_t}= s (1-\sigma) \frac{k_t^{\alpha}}{k_t} -( n + \delta ) $$

現在解決上述穩態 $ \frac{k_{t+1} - k_t }{k_t}=0 $ (根據定義一個穩定的狀態),給我們:

$$ 0 = s (1-\sigma) \frac{k_t^{\alpha}}{k_t} -( n + \delta ) $$

最終可以求解最優穩態人均資本:

$$ k^* = \left( \frac{s(1-\sigma)}{n+\delta} \right)^{\frac{1}{1-\alpha}} $$

此外,Barro & Sala-i-Martin Economic Growth 第 2 版涵蓋了與該模型非常相似的模型。因此,您可以查看它以獲取更多詳細資訊。

引用自:https://economics.stackexchange.com/questions/41174